Quantcast
  • Register
PhysicsOverflow is a next-generation academic platform for physicists and astronomers, including a community peer review system and a postgraduate-level discussion forum analogous to MathOverflow.

Welcome to PhysicsOverflow! PhysicsOverflow is an open platform for community peer review and graduate-level Physics discussion.

Please help promote PhysicsOverflow ads elsewhere if you like it.

News

PO is now at the Physics Department of Bielefeld University!

New printer friendly PO pages!

Migration to Bielefeld University was successful!

Please vote for this year's PhysicsOverflow ads!

Please do help out in categorising submissions. Submit a paper to PhysicsOverflow!

... see more

Tools for paper authors

Submit paper
Claim Paper Authorship

Tools for SE users

Search User
Reclaim SE Account
Request Account Merger
Nativise imported posts
Claim post (deleted users)
Import SE post

Users whose questions have been imported from Physics Stack Exchange, Theoretical Physics Stack Exchange, or any other Stack Exchange site are kindly requested to reclaim their account and not to register as a new user.

Public \(\beta\) tools

Report a bug with a feature
Request a new functionality
404 page design
Send feedback

Attributions

(propose a free ad)

Site Statistics

205 submissions , 163 unreviewed
5,047 questions , 2,200 unanswered
5,345 answers , 22,709 comments
1,470 users with positive rep
816 active unimported users
More ...

  Is it possible that a particle is much heavier through a loop correction?

+ 2 like - 0 dislike
505 views

Let's assume, we have standard model singlet particle $s$, that mixes after electroweak symmetry breaking with an exotic, vectorlike neutral lepton $N$. The relevant part of the Lagrangian reads

$$ L \supset h^c s N + h s N^c + M N N^c, $$

where $h$ is the standard model higgs and $M$ is a superheavy mass. Moreover, we assume that for some reason there is (at tree level) no Majorana mass term: $ M_s ss$ for the singlet $s$. The tree-level analysis now yields for the singlet $s$ a tiny seesaw type mass: $m_s \approx v_{EW} / M^2 \ll v_{EW}$. 

Now, a Majorana mass term for the singlet $s$ will be generically generated at the 1-loop level through a diagram with $N$ in the loop. It was pointed out to me that this 1-loop contribution "may give rise to a much larger mass for the singlet". I would like understand how this can happen. I think the relevant diagram looks like this

https://physics.stackexchange.com/questions/363171/is-it-possible-that-are-particle-is-much-heavier-through-a-loop-correction

My naive estimate for this one-loop contribution is $ m_s \approx 1/16 \pi \ m_{EW}^2 /M$, i.e. something comparable to the tree-level estimate, divided by a loop factor, possibly times some logarithm. Thus, while there is possibly some relevant correction due to the logarithm, the consequences do not seem dramatic.

Is there any other possible correction that I'm missing here? Is there some diagram that potentially leads to a much heavier mass for the singlet $s$?


----------


A relevant analogous scenario

The situation is similar to the usual seesaw for the left-handed neutrinos $\nu_L$. However, the situation described above is reversed. In the usual seesaw scenario, the left-handed neutrinos $\nu_L$ are light and the singlet $\nu_R$ is heavy. The 1-loop correction to the usual seesaw formula, is discussed in On the importance of the 1-loop finite corrections to seesaw neutrino masses by D. Aristizabal Sierra, Carlos E. Yaguna. (See also, this summary. The relevant diagrams are

Source: http://iopscience.iop.org/article/10.1088/1742-6596/375/1/042003/meta

and the result is

This result yields a contribution comparable to the result of the tree-level analysis: $ m_{EW}^2 / M$, where $m_{EW}$ denotes the electroweak scale and $M$ a superheavy scale. (In addition the is a potentially enhancing log factor.)
 

 

asked Oct 16, 2017 in Phenomenology by JakobS (110 points) [ revision history ]

Your answer

Please use answers only to (at least partly) answer questions. To comment, discuss, or ask for clarification, leave a comment instead.
To mask links under text, please type your text, highlight it, and click the "link" button. You can then enter your link URL.
Please consult the FAQ for as to how to format your post.
This is the answer box; if you want to write a comment instead, please use the 'add comment' button.
Live preview (may slow down editor)   Preview
Your name to display (optional):
Privacy: Your email address will only be used for sending these notifications.
Anti-spam verification:
If you are a human please identify the position of the character covered by the symbol $\varnothing$ in the following word:
p$\hbar$ysicsOve$\varnothing$flow
Then drag the red bullet below over the corresponding character of our banner. When you drop it there, the bullet changes to green (on slow internet connections after a few seconds).
Please complete the anti-spam verification




user contributions licensed under cc by-sa 3.0 with attribution required

Your rights
...